Difference between revisions of "2022 AMC 10A Problems/Problem 9"

(Solution)
(Solution)
Line 13: Line 13:
 
==Solution==
 
==Solution==
 
The top left rectangle can be <math>5</math> possible colors. Then the bottom left region can only be <math>4</math> possible colors, and the bottom middle can only be <math>3</math> colors since it is next to the top left and bottom left. Similarly, we have <math>3</math> choices for the top right and <math>3</math> choices for the bottom right, which gives us a total of <math>5\cdot4\cdot3\cdot3\cdot3=\boxed{\textbf{(D) }540}</math>.
 
The top left rectangle can be <math>5</math> possible colors. Then the bottom left region can only be <math>4</math> possible colors, and the bottom middle can only be <math>3</math> colors since it is next to the top left and bottom left. Similarly, we have <math>3</math> choices for the top right and <math>3</math> choices for the bottom right, which gives us a total of <math>5\cdot4\cdot3\cdot3\cdot3=\boxed{\textbf{(D) }540}</math>.
 +
 
~Txu
 
~Txu
  

Revision as of 01:13, 12 November 2022

A rectangle is partitioned into 5 regions as shown. Each region is to be painted a solid color - red, orange, yellow, blue, or green - so that regions that touch are painted different colors, and colors can be used more than once. How many different colorings are possible? [asy] size(5.5cm); draw((0,0)--(0,2)--(2,2)--(2,0)--cycle); draw((2,0)--(8,0)--(8,2)--(2,2)--cycle); draw((8,0)--(12,0)--(12,2)--(8,2)--cycle); draw((0,2)--(6,2)--(6,4)--(0,4)--cycle); draw((6,2)--(12,2)--(12,4)--(6,4)--cycle); [/asy]

$\textbf{(A) }120\qquad\textbf{(B) }270\qquad\textbf{(C) }360\qquad\textbf{(D) }540\qquad\textbf{(E) }720$

Solution

The top left rectangle can be $5$ possible colors. Then the bottom left region can only be $4$ possible colors, and the bottom middle can only be $3$ colors since it is next to the top left and bottom left. Similarly, we have $3$ choices for the top right and $3$ choices for the bottom right, which gives us a total of $5\cdot4\cdot3\cdot3\cdot3=\boxed{\textbf{(D) }540}$.

~Txu

See Also

2022 AMC 10A (ProblemsAnswer KeyResources)
Preceded by
Problem 8
Followed by
Problem 10
1 2 3 4 5 6 7 8 9 10 11 12 13 14 15 16 17 18 19 20 21 22 23 24 25
All AMC 10 Problems and Solutions

The problems on this page are copyrighted by the Mathematical Association of America's American Mathematics Competitions. AMC logo.png